Đến nội dung

githenhi512 nội dung

Có 276 mục bởi githenhi512 (Tìm giới hạn từ 06-06-2020)



Sắp theo                Sắp xếp  

#628972 Tìm m để khoảng cách từ O đến đường thẳng (dm) là lớn nhất

Đã gửi bởi githenhi512 on 22-04-2016 - 20:27 trong Đại số

Cho đường thẳng (m+2)x - my = -1 (dm) (m là tham số)

Tìm m để khoảng cách từ O đến đường thẳng (dm) là lớn nhất 

cho x=0 $\Rightarrow y=\frac{1}{m}\Rightarrow A(0;\frac{1}{m})\in Oy$

cho y=0 $\Rightarrow x=\frac{-1}{m+2}\Rightarrow B(\frac{-1}{m+2};0)\in Ox$

Kẻ OH vuông góc vs AB. Tg AOB v tại A

 $\Rightarrow \frac{1}{OH^{2}}=\frac{1}{OA^{2}}+\frac{1}{OB^{2}}$

                                               =$m^{2}+(m+2)^{2}=2(m+1)^{2}+2\geq 2$

$\Rightarrow$ Max OH=2 $\Leftrightarrow$ x=-1




#628982 Tìm $MinA=\frac{a^3}{2a^2+b^2}$

Đã gửi bởi githenhi512 on 22-04-2016 - 20:57 trong Bất đẳng thức và cực trị

Ở đây nha bạn!

http://mathvn.net/fo...=784&rowstart=5

P/s: Mình cũng k hiểu lắm :wacko:




#628999 min $P=2(a+b+c)+\dfrac{1}{a}+\dfrac{1}{b}+\dfrac{1}{c}$

Đã gửi bởi githenhi512 on 22-04-2016 - 21:21 trong Bất đẳng thức và cực trị

$(a^{2}+b^{2}+c^{2})(ab+bc+ca)^{2}\leq \frac{(a+b+c)^{6}}{27}$ la sao vay??

Áp dụng bđt Cauchy $abc\leq \frac{(a+b+c)^{3}}{27}$ tc: 

$VT=(\sum a^{2}).(\sum ab).(\sum ab)\leq \frac{(\sum a^{2}+2(\sum ab))^{3}}{27}=VP$




#629024 TOPIC luyện thi vào lớp 10 chuyên toán năm 2016 - 2017.

Đã gửi bởi githenhi512 on 22-04-2016 - 22:02 trong Tài liệu - Đề thi

           $\left\{\begin{matrix} (x+\sqrt{x^{2}+1})(y+\sqrt{y^{2}+1})=1 & & \\ 3x^{2}+y+3=\sqrt{3x+1}+\sqrt{4-5y}& & \end{matrix}\right.$

Đk: $x\geq \frac{-1}{3},y\leq 0.8$

Từ (1). Mà $(y+\sqrt{y^{2}+1})(\sqrt{y^{2}+1}-y)=1$, $(\sqrt{x^{2}+1}+x)(\sqrt{x^{2}+1}-x)=1$

$\Rightarrow x=-y$ thay vào (2) tđ:

$3x^{2}-x+3=\sqrt{3x+1}+\sqrt{4+5x}$

$\Leftrightarrow 3x(x-1)=\left [ \sqrt{3x+1} -(x+1)\right ]+\left [ \sqrt{4+5x}-(x+2) \right ]$

$\Leftrightarrow 3x(x-1)=\frac{x(1-x)}{\sqrt{3x+1}+x+1}+\frac{x(1-x)}{\sqrt{4+5x}+x+2}$

Mà $3+\frac{1}{\sqrt{3x+1}+x+1}+\frac{1}{\sqrt{4+5x}+x+2}> 0\vee x\geq \frac{-1}{3}$

$\Rightarrow x(x-1)=0$

N x=0(t/m) $\Rightarrow$ y=0(t/m)

N x=1(t/m) $\Rightarrow$ y=-1(t/m)




#629030 $x^{4}-4x^{3}+x^{2}+4x+1=0$

Đã gửi bởi githenhi512 on 22-04-2016 - 22:17 trong Phương trình, hệ phương trình và bất phương trình

có phương pháp gì mà phân tích đk như thế k bạn

x=0 k là no pt nên x khác 0

$\Rightarrow x^{2}-4x+1+\frac{4}{x}+\frac{1}{x^{2}}=0$

$\Leftrightarrow (x-\frac{1}{x})^{2}-4(x-\frac{1}{x})+3=0$

$\Rightarrow x-\frac{1}{x}\in \left \{ 1;3 \right \}$




#629400 Chứng minh rằng: $\frac{x}{x+\sqrt{3x+yz}}+\frac{y}{y+...

Đã gửi bởi githenhi512 on 24-04-2016 - 20:23 trong Bất đẳng thức và cực trị

$cho x,y,z là ba số dương thỏa mãn x+y+z=3. chứng minh rằng: \frac{x}{x+\sqrt{3x+yz}}+\frac{y}{y+\sqrt{3y+zx}}+\frac{z}{z+\sqrt{3z+xy}}\leq 1$

Tc: $\sqrt{(x+y)(x+z)}\leq \sqrt{xy}+\sqrt{xz}$(x,z>0)

$\frac{x}{x+\sqrt{3x+yz}}=\frac{x}{x+\sqrt{(x+y+z)x+yz}}=\frac{x}{x+\sqrt{(x+y)(x+z)}}\leq \frac{x}{x+\sqrt{xy}+\sqrt{xz}}=\frac{\sqrt{x}}{\sqrt{x}+\sqrt{y}+\sqrt{z}}$

Tương tự tc đpcm

Dấu ''='' xr khi x=y=z=1




#629415 Giải hệ phương trình: $$\left\{\begin{matrix} \...

Đã gửi bởi githenhi512 on 24-04-2016 - 21:08 trong Phương trình - hệ phương trình - bất phương trình

Giải hệ phương trình:

$\left\{\begin{matrix} \sqrt{ab-6}=12-b^2 \\ ab=3+a^2 \end{matrix}\right.$

ĐK: $ab\geq 6$

Từ (2) $\Rightarrow a^{2}-ab+3=0. \Delta =b^{2}-12$

$\Rightarrow b^{2}-12\geq 0$

Lại có từ (1) $\Rightarrow b^{2}\leq 12$ $\Rightarrow b^{2}=12\Rightarrow b\left \{ -\sqrt3.{2} ;2.\sqrt{3}\right \}$




#629430 Tìm tất cả các số nguyên tố a,b,c thõa mãn: $a^{b}+b^{a...

Đã gửi bởi githenhi512 on 24-04-2016 - 21:40 trong Số học

Tìm tất cả các số nguyên tố a,b,c thõa mãn: $a^{b}+b^{a}=c$

$a,b\geq 2\Rightarrow c>2\Rightarrow c lẻ$ $\Rightarrow a, b k cùng tính chẵn lẻ$

Do vai trò của a,b là như nhau. K mất tính tổng quat gs b chẵn $\Rightarrow b=2\Rightarrow a^{2}+2^{a}=c$

N a=3 $\Rightarrow c=17(t/m)$

N $a\neq 3\Rightarrow a^{2}=3k+1(k\in \mathbb{N}, \neq 0)$

Mà $2^{a}=(3-1)^{a}=3n-1(a lẻ)$$(n\in \mathbb{N},\neq 0)$

$\Rightarrow c=3(k+n)(l)$

Vậy (a,b,c)=(2;3;17);(3;2;17)




#629436 CMR : $\sum \sqrt{3+4^{x}}\geqslant 6...

Đã gửi bởi githenhi512 on 24-04-2016 - 21:57 trong Bất đẳng thức và cực trị

Cho x;y;z là 3 số thực thỏa mãn x+y+z=0 . CMR : $\sum \sqrt{3+4^{x}}\geqslant 6$

$\sqrt{3+4^{x}}=\sqrt{1+1+1+4^{x}}\geq \sqrt{4.\sqrt[4]{4^{x}}}=2.\sqrt[4]{2^{x}}$

Tương tự $\Rightarrow VT\geq 2(\sum \sqrt[4]{2^{x}})\geq 2.3.\sqrt[12]{2^{x+y+z}}=6(đpcm)$

Dấu ''='' xr khi x=y=z=0




#629444 Cho tam giác ABC. Chứng minh: $(b+c)cosA+(c+a)cosB+(a+b)cosC=a+b+c$

Đã gửi bởi githenhi512 on 24-04-2016 - 22:39 trong Hình học

Cho tam giác ABC. Chứng minh:

$(b+c)cosA+(c+a)cosB+(a+b)cosC=a+b+c$

Áp dụng đ/l hàm số Cos trong tam giác ta có: $CosA=\frac{b^{2}+c^{2}-a^{2}}{2bc}$

$\Rightarrow (b+c).CosA=\frac{(b^{2}+c^{2}-a^{2})(b+c)}{2bc}$

Tượng tự tc: $VT=\frac{1}{2}.\frac{(b^{2}+c^{2}-a^{2})(b+c).a+(c^{2}+a^{2}-b^{2})(c+a).b+(a^{2}+b^{2}-c^{2})(a+b).c}{abc}$

                          =$\frac{1}{2}.\frac{2abc(a+b+c)}{abc}=a+b+c$(đpcm)




#629448 chứng minh

Đã gửi bởi githenhi512 on 24-04-2016 - 22:56 trong Bất đẳng thức và cực trị

$cho 2015 số nguyên dương a1,a2,..a2015 thỏa mãn điều kiện: \frac{1}{\sqrt{a1}}+\frac{1}{\sqrt{a2}}+\frac{1}{\sqrt{a3}}+...+\frac{1}{\sqrt{a2015}}\geq 89 chứng minh rằng trong số nguyên dương đó, luôn tồn tại ít nhất 2 số bằng nhau$

Gs (.) 2015 số trên k tồn tại 2 số bằng nhau và $a_{1}<a_{2}< ...< a_{2015}\Rightarrow a_{1}\geq 1, a_{2}\geq 2,.., a_{2015}\geq 2015$

$\Rightarrow VT<\frac{1}{1}+\frac{1}{$\sqrt{2}$}+...+\frac{1}{$\sqrt{2015}$}$

Lại có: $\frac{1}{\sqrt{k}}=\frac{2}{\sqrt{k}}<\frac{2}{\sqrt{k}+\sqrt{k-1}}=2(\sqrt{k}-\sqrt{k-1})(k\in \mathbb{N}, \neq 0)$

Dđ: VT$<1+2(\sqrt{2}-\sqrt{1}+\sqrt{3}-\sqrt{2}+...+\sqrt{2015}-\sqrt{2014})=2\sqrt{2015}-1<89(l)$

$\Rightarrow điều gs là sai \Rightarrow đpcm$




#629451 $n^2+1$ tăng

Đã gửi bởi githenhi512 on 24-04-2016 - 23:04 trong Tổ hợp và rời rạc

https://www.wattpad....-toán-rời-rạc-1




#629456 $\sum \frac{\left ( b+c-a \right )^{2}}{\left (...

Đã gửi bởi githenhi512 on 24-04-2016 - 23:18 trong Bất đẳng thức và cực trị

http://diendantoanho...uage=1&langid=1




#629459 Chứng minh K thuộc một đường thẳng cố định khi d thay đổi và thỏa mãn điều ki...

Đã gửi bởi githenhi512 on 24-04-2016 - 23:52 trong Hình học

Bạn có thể tham khảo ở đây!

http://d.violet.vn/b...ntry_id/9389980




#629551 GPT $\sqrt{4x^2+5x+1}-2\sqrt{x^2-x+1}=3-9x...

Đã gửi bởi githenhi512 on 25-04-2016 - 20:00 trong Phương trình, hệ phương trình và bất phương trình

GPT

$\sqrt{4x^2+5x+1}-2\sqrt{x^2-x+1}=3-9x$

Đặt $\sqrt{4x^{2}+5x+1}=a\geq 0, \sqrt{x^{2}-x+1}=b>0\Rightarrow a-2b=4b^{2}-a^{2}\Rightarrow a=2b$




#629579 CMR$\sum \frac{b^{3}}{a^{2}...

Đã gửi bởi githenhi512 on 25-04-2016 - 21:08 trong Bất đẳng thức và cực trị

Cho a,b,c>0.CMR$\sum \frac{b^{3}}{a^{2}(a^{3}+2b^{3})}\geq \frac{1}{3}(\sum \frac{1}{a^{2}})$

$\frac{b^{3}}{a^{2}.(a^{3}+2b^{3})}=\frac{1}{2}.(\frac{1}{a^{2}}-\frac{a}{a^{3}+2b^{3}})= \frac{1}{2}.(\frac{1}{a^{2}}-\frac{a}{a^{3}+a^{3}+b^{3}})\geq \frac{1}{2}(\frac{1}{a^{2}}-\frac{1}{3ab})$

Tương tự tc: VT $\geq \frac{1}{2}.(\sum \frac{1}{a^{2}}-\frac{1}{3}.\sum \frac{1}{ab})\geq VT$(đpcm)




#629718 giải phương trình $\frac{1}{\sqrt{x^{...

Đã gửi bởi githenhi512 on 26-04-2016 - 21:08 trong Phương trình, hệ phương trình và bất phương trình

câu hệ có sai đề k bn?




#629721 giải phương trình $\frac{1}{\sqrt{x^{...

Đã gửi bởi githenhi512 on 26-04-2016 - 21:15 trong Phương trình, hệ phương trình và bất phương trình

ĐK x>-1 mà




#629727 giải phương trình $\frac{1}{\sqrt{x^{...

Đã gửi bởi githenhi512 on 26-04-2016 - 21:32 trong Phương trình, hệ phương trình và bất phương trình

(1)+(2)$\Rightarrow x^{3}+y^{3}+6xy-8=0\Leftrightarrow 0,5.(x+y-2)\left [ (x-y) ^{2}+(x+2)^{2}+(y+2)^{2}\right ]=0$

N x=y=-2 thay vào (2) k t/m(l)

N x+y=2 $\Rightarrow x=2-y$ thế vào (2) $\Rightarrow 7y^{2}-12y+5=0$

$\Rightarrow (y-1)(7y-9)=0$




#629728 $Sin6a . Cot3a - Cos6a$

Đã gửi bởi githenhi512 on 26-04-2016 - 21:43 trong Công thức lượng giác, hàm số lượng giác

http://123doc.org/do...am-chia-cot.htm




#629748 Chứng minh: IE // BC

Đã gửi bởi githenhi512 on 26-04-2016 - 22:35 trong Hình học

Rất mong các bạn gợi ý giúp! Chỉ cần gợi ý thôi không cần giải chi tiết! (Sợ làm phiền các bạn) Cám ơn nhiều luôn!

Câu 2*: Cho đường tròn tâm (O) và điểm M nằm ngoài đường tròn (O). Đường thẳng MO cắt (O) tại E và F (ME < MF). Vẽ cát tuyến MAB và tiếp tuyến MC của (O) (C là tiếp điểm, A nằm giữa hai điểm M và B, A và C khác phía đối với đường thẳng MO). Trên nửa mặt phẳng bời OM có chứa điểm A, vẽ nửa đường tròn đường kính MF, nửa đường tròn này cắt tiếp tuyến tại E của (O) ở K. Gọi S là giao điểm của hai đường thẳng CO và KF.

a) Chứng minh đường thẳng MS vuông góc KC.

b) Gọi P, Q lần lượt là tâm đường tròn ngoại tiếp ∆EFS, ∆ABS và T là trung điểm của KS. Chứng minh ba điểm P, Q, T thẳng hàng.

Hình vẽ: https://uphinhnhanh....4_Picture22.png

51074_Picture22.png

a. $MK^{2}=ME.MF=MC^{2}\Rightarrow MK=MC$

Mà TG MKSC nt$\Rightarrow đpcm$

b. Gọi G là gđ của SM và CK

Tg GCK v tại G, trung tuyến GT nên TS=TG

Mà MG.MS=ME.MF(=$MC^{2}$)$\Rightarrow EGSF nt$

Tương tự SGAB nt $\Rightarrow$ SG là dây chung của (P) và (Q) 

$\Rightarrow$ PQ là trung trực của SG $\Rightarrow$ đpcm




#630087 TOPIC luyện thi vào lớp 10 chuyên toán năm 2016 - 2017.

Đã gửi bởi githenhi512 on 28-04-2016 - 20:44 trong Tài liệu - Đề thi

chém bài cho vui : (Tuyển sinh lớp 10 Quốc Học Huế, năm 2008 - 2009 - Chuyên toán)

Cho phương trình: $x^{4}-2mx^{2}+2m-1=0$. Tìm giá trị m để phương trình có bốn nghiệm $x_{1},x_{2},x_{3},x_{4}$ sao cho:

$x_{1}<x_{2}<x_{3}<x_{4}$ và $x_{4}-x_{1}=3(x_{3}-x_{2})$.

Đặt x2=y$\geq 0\Rightarrow y^{2}-2my+2m-1=0$(1)

Gọi y1, y2 là 2 no của (1) $\Rightarrow y_{1}+y_{2}=2m, y_{1}.y_{2}=2m-1$

Mà x1<x2<x3<x4 $\Rightarrow \sqrt{y_{1}}=-x_{1}=x_{4}, \sqrt{y_{2}}=x_{3}=-x_{2}$

Lại có $x_{4}-x_{1}=3(x_{3}-x_{2})\Rightarrow 2\sqrt{y_{1}}=6\sqrt{y_{2}}\Rightarrow y_{1}=9y_{2}$

$\Rightarrow 9y_{2}=2m-1,10y_{2}=2m\Rightarrow y_{2}=\frac{m}{5} thay  vào 9y_{2}=2m-1  tđ:9m^{2}-50m+25=0$

$\Rightarrow m\in \left \{ \frac{5}{9} ;5\right \}$

 




#630137 Tìm a,b biết $x=\frac{\sqrt{5}-\sqrt{3}}{\sqrt{5}+...

Đã gửi bởi githenhi512 on 28-04-2016 - 22:54 trong Phương trình, hệ phương trình và bất phương trình

cho phương trình $ax^{2}+bx+1=0$,với a,b là các số hữu tỷ.
Tìm a,b biết $x=\frac{\sqrt{5}-\sqrt{3}}{\sqrt{5}+\sqrt{3}}$ là nghiệm của phương trình

$\Leftrightarrow x=\frac{(\sqrt{5}-\sqrt{3})^{2}}{2}=4-\sqrt{15}$

$\Leftrightarrow (4-\sqrt{15})^{2}.a+(4-\sqrt{15}).b+1=0\Leftrightarrow (31a+4b+1)-\sqrt{15}.(8a+b)=0$

Mà a, b hữu tỷ $\Rightarrow 31a+4b+1=8a+b=0\Rightarrow ..$




#630782 CM: $\frac{a+3c}{a+b}+\frac{c+3a...

Đã gửi bởi githenhi512 on 02-05-2016 - 15:46 trong Bất đẳng thức và cực trị

1. Cho a,b,c>0. CM: $\frac{a+3c}{a+b}+\frac{c+3a}{b+c}+\frac{4b}{c+a}\geq 6$

2. Cho a,b,c>0 t/m: ab+bc+ca=3.CM: $\sum \frac{a}{2a^{2}+bc}\geq abc$




#630791 Tìm GTNN của P

Đã gửi bởi githenhi512 on 02-05-2016 - 16:09 trong Bất đẳng thức và cực trị

Cho a,b>0

tìm GTNN của P=$\frac{a^{2}+b^{2}}{ab}+\frac{\sqrt{ab}}{a+b}$

P=$\frac{1}{4}.\frac{a^{2}+b^{2}}{ab}+\frac{\sqrt{ab}}{a+b}+\frac{3}{4}.\frac{a^{2}+b^{2}}{ab}\geq \frac{(a+b)^{2}}{8ab}+\frac{\sqrt{ab}}{a+b}+\frac{3}{4}.2$

 $\geq 2\sqrt{\frac{a+b}{8\sqrt{ab}}}+1.5\geq 2\sqrt{\frac{2}{8}}+1.5=2.5$

$\Rightarrow Min P=2.5\Leftrightarrow a=b>0$